You are on page 1of 98

12/16/21, 1:39 PM Assignment Print View

 
 1.    

Ferris Company has an old machine that is fully depreciated but has a current salvage value of
$10,000. The company wants to purchase a new machine that would cost $60,000 and have a five-
year useful life and zero salvage value. Expected changes in annual revenues and expenses if the
new machine is purchased are:
 
Increased revenues $10,000 $120,000
Increased expenses: 14,000  
Salary of additional operator    
Supplies    
Depreciation 12,000    
Maintenance 8,000 90,000
Increased net income   $30,000

(Ignore income taxes in this problem.)

Required:

a) What is the payback period on the new equipment?


b) What is the simple rate of return on the new equipment?

a) Investment required/Net annual cash inflow = Payback period


($60,000 - $10,000) / ($30,000 + 18,000) = 1.04 years

b) Incremental net income/Investment = Simple rate of return


$30,000 / $70,000 = 42.86%

https://ezto.mheducation.com/hm.tpx 1/98
12/16/21, 1:39 PM Assignment Print View

 
 2.    

Which of the following capital budgeting techniques consider(s) cash flow over the entire life of the
project?
 
Internal Rate of Return Payback
A) Yes Yes
B) Yes No
C) No Yes
D) No No

 Option A

 Option B

 Option C

 Option D

 
 3.    

A company is considering updating its obsolete plant and equipment. The new equipment would
be assumed to have a ten-year useful life. A discounted cash flow analysis of the costs and benefits
showed that the plant and equipment would have a net present value of ($452,000). However,
uncertainty exists as to the intangible benefits of upgrading the plant and equipment. The company
is subject to a cost of capital of 12%. How much would the plant and equipment's intangible benefits
have to amount to annually to make the investment worthwhile?

 $40,000

 $80,000

 $20,000

 $32,563

PVA (10 years, 12%): 5.650.


 
Additional cash flow per year required from intangible benefits = net present value to
be offset / present value factor = $452,000 / 5.650 = $80,000 annually.

https://ezto.mheducation.com/hm.tpx 2/98
12/16/21, 1:39 PM Assignment Print View

 
 4.    

(Appendix 13A) Which of the following would decrease the net present value of a project?

 A decrease in the income tax rate.

 A decrease in the initial investment.

 An increase in the useful life of the project.

 An increase in the discount rate.

 
 5.    

Projects can be compared using the NPV method regardless of the required investment.

 True

 False

Projects can only be compared if they have equal investments.

https://ezto.mheducation.com/hm.tpx 3/98
12/16/21, 1:39 PM Assignment Print View

 
 6.    

A finance director must choose one among four of the following projects:
 
Project Net Present Value Initial Investment
A $ 200,000 $ 100,000
B $ 160,000 $ 90,000
C $ 200,000 $ 150,000
D $ 180,000 $ 54,000
 
Which of the above projects should the director choose first?

 A

 B

 C

 D

Project profitability index = present value of net cash inflows / investment required.
 
Project profitability index (A) = $200,000 / $100,000 = 2.00.
Project profitability index (B) = $160,000 / $90,000 = 1.78.
Project profitability index (C) = $200,000 / $150,000 = 1.33.
Project profitability index (D) = $180,000 / $54,000 = 3.33.
 
Project D has the highest profitability index, so it should be the director’s first choice.

https://ezto.mheducation.com/hm.tpx 4/98
12/16/21, 1:39 PM Assignment Print View

 
 7.    

(Appendix 13A) The following information is available on a new piece of equipment:


 
Cost of the Equipment $21,720
Annual Cash Inflows 5,000
Internal Rate of Return 16%
Required Rate of Return 10%

What is approximately the life of the equipment? (Ignore income taxes in this problem.)

 4.3 years.

 6.0 years.

 8.0 years.

 It is impossible to determine from the data given.

Calculated using the NPER formula of Microsoft Excel at 16%.

 
 8.    

(Appendix 13B) The release of working capital at the end of an investment project is a taxable cash
inflow.

 True

 False

https://ezto.mheducation.com/hm.tpx 5/98
12/16/21, 1:39 PM Assignment Print View

 
 9.    

A company is considering buying a new machine for $10,000. The machine is expected to save the
company $6,000 for each of the next 5 years. The cost of the machine will be depreciated evenly
over the next 5 years, with no salvage value. Compute the machine's simple rate of return. Ignore
income taxes.

 30%

 10%

 40%

 20%

Sample rate of return = incremental operating revenues / initial investment = (incremental revenues
- incremental expenses including depreciation) / initial investment = ($6,000 - ($10,000 - 5)) /
$10,000 = 40%

 
 10.    

A company is considering updating its obsolete plant and equipment. The new equipment would
be assumed to have a twenty-five year useful life. A discounted cash flow analysis of the costs and
benefits showed that the plant and equipment would have a net present value of ($800,000).
However, uncertainty exists as to the intangible benefits of upgrading the plant and equipment. The
company is subject to a cost of capital of 8%. How much would the plant and equipment’s
intangible benefits have to amount to annually to make the investment worthwhile?

 $37,471

 $20,000

 $47,931

 $79,941

PVA (25 years, 8%): 10.675.


Additional cash flow per year required from intangible benefits = net present value to be offset /
present value factor = $800,000 / 10.675 = $79,941 annually (rounded).

https://ezto.mheducation.com/hm.tpx 6/98
12/16/21, 1:39 PM Assignment Print View

 
 11.    

A post audit involves evaluating the extent to which expected results are actually being realized.

 True

 False

 
 12.    

A company is considering buying a new machine for $50,000. The machine is expected to save the
company $25,000 for each of the next 5 years. The cost of the machine will be depreciated evenly
over the next 5 years, with no salvage value. Compute the machine's simple rate of return. Ignore
income taxes.

 20%

 8%

 2%

 30%

Simple rate of return = incremental operating revenues / initial investment = (incremental revenues
– incremental expenses including depreciation) / initial investment = ($25,000 – ($50,000 / 5)) /
$50,000 = 30%.

https://ezto.mheducation.com/hm.tpx 7/98
12/16/21, 1:39 PM Assignment Print View

 
 13.    

(Appendix 13A) In order to receive $12,000 at the end of three years and $10,000 at the end of five
years, how much must be invested now if you can earn 14% rate of return? (Ignore income taxes in
this problem.) (Round your PV factor to 5 decimal places and final answer to nearest whole dollar
amount.)

 $8,100

 $12,978

 $13,293

 $32,054

Calculated using the PV formula of Microsoft Excel.

 
 14.    

A company is considering updating its obsolete plant and equipment. The new equipment would
be assumed to have a fifteen-year useful life. A discounted cash flow analysis of the costs and
benefits showed that the plant and equipment would have a net present value of ($240,000).
However, uncertainty exists as to the intangible benefits of upgrading the plant and equipment. The
company is subject to a cost of capital of 14%. How much would the plant and equipment's
intangible benefits have to amount to annually to make the investment worthwhile?

 $10,000

 $20,000

 $30,000

 $39,072

PVA (15 years, 14%): 6.142.


 
Additional cash flow per year required from intangible benefits = net present value to
be offset / present value factor = $240,000 / 6.142 = $39,072 annually (rounded).

https://ezto.mheducation.com/hm.tpx 8/98
12/16/21, 1:39 PM Assignment Print View

 
 15.    

In comparing two investment alternatives, the difference between the net present values of the two
alternatives obtained using the total cost approach will be the same as the net present value
obtained using the incremental cost approach.

 True

 False

 
 16.    

When using the internal rate of return method to evaluate investment projects, if the internal rate of
return is less than the required rate of return, the project should be accepted.

 True

 False

https://ezto.mheducation.com/hm.tpx 9/98
12/16/21, 1:39 PM Assignment Print View

 
 17.    

(Appendix 13A) The following data pertain to an investment proposal:


 
Investment in the Project (equipment) $14,000
Net Annual Cash Inflows Promised $2,800
Working Capital Required $5,000
Salvage Value of the Equipment $1,000
Life of the Project 10 years

The working capital would be released for use elsewhere when the project is completed. What is
the net present value of the project, using a discount rate of 8%? (Ignore income taxes in this
problem.) (Do not round your intermediate calculations and round the final answer to the nearest
whole dollar.)

 ($251)

 $251

 $2,567

 $5,251

Calculated using the NPV formula of Microsoft Excel.

https://ezto.mheducation.com/hm.tpx 10/98
12/16/21, 1:39 PM Assignment Print View

 
 18.    

(Appendix 13A and 13B) Vernal Company has been offered a seven-year contract to supply a part
for the military. After careful study, the company has developed the following estimated data
relating to the contract:
 
Cost of equipment needed $300,000
Working capital needed to carry inventories $50,000
Annual Net Cash Inflow $90,000
Salvage Value of Equipment $10,000

The equipment above would be in Class 7 with a 15% CCA rate. The company would take the
maximum CCA allowable each year. It is not expected that the contract would be extended beyond
the initial contract period. The company's after-tax cost of capital is 10%, and the tax rate is 30%.

Required:

Use net present value analysis to determine whether or not the contract should be accepted.
(Round all calculations to the nearest dollar.)

The following is one approach to answering the question.


 
  Year Cash Flow Tax Effect After-tax Cash 10% Factor Present Value
Flow
Cost of Now $(300,000) 1.00 $(300,000) 1.00 $(300,00)
equipment
Working Now $(50,000) 1.00 $(50,000) 1.00 $(50,000)
capital
needed
Net annual 1-7 $90,000 0.70 $63,000 4.868 $306,684
cash inflows
Salvage value 7 $10,000 0.70 $7,000 0.513 $3,591
Working 7 $50,000 0.00 $50,000 0.513 $25,650
capital
released
PV of CCA tax           $50,619
shield
Net present           $10,894
value

NPV of cash flows = $(12,500.13). Add to this the present value of CCA tax savings shown below of
$50,619** the NPV of the project = $38,118.87. Calculated using the NPV formula of Microsoft Excel
Decision: The contract should be accepted, because the project has a positive net present value.
* Because the equipment was acquired for this specific contract, one might assume there will be no
balance in the Class and therefore the proceeds from sale of the equipment will be recaptured and
fully taxed as ordinary income at the 30% tax rate. Since all examples in the text consider the asset
class remaining open and most students will therefore make the assumption there are still assets in
this class the solution does not tax the sale of the equipment at the end. If using the after-tax
amount the NPV of the project is reduced by $1,540. Students who refer to their finance courses
might consider the sale of asset as a capital gain taxed differently.

https://ezto.mheducation.com/hm.tpx 11/98
12/16/21, 1:39 PM Assignment Print View

** Calculating the present value (PV) of the CCA tax shield.


First component (assuming zero salvage value):
 
PV =  Cdt   ×  (1 + 0.5k)
         (d+k)        (1 + k)

      = (300,000 × .15 × .30) × (1 + 0.5 × .10)


                  (.15 + .10)                (1 + .10)

      = $54,000 × 0.9545

      =$51,543

Second component (correction for non-zero salvage value):


 
PV = (S × d × t) × (1 + k)-n
            (d + k)

      = (10,000 × .15 × .30 × (1 + 10)-7


          (.15 + .10)
      = $1,800 × 0.513
      = $924
Total present value = $51,543 - $924
      = $50.619

 
 19.    

(Appendix 13B) A company had tax-deductible cash expenses of $900,000 last year, and the tax
rate was 25%. What was the after-tax net cash outflow for these expenses?

 $195,000.

 $390,000.

 $675,000.

 $650,000.

$900,000 *(1-0.25) = $675,000.

https://ezto.mheducation.com/hm.tpx 12/98
12/16/21, 1:39 PM Assignment Print View

 
 20.    

What is the present value of a 10-year, $12,000 after-tax annual cash flow stream at 10%? Assume
the cash flows occur at the start of each period.

 $50,000

 $61,450

 $80,000

 $81,108

We evaluate this as a single payment of 12,000 plus a 9 year ordinary annuity, $12,000 + (12,000 x
5.759) = $81,108.

 
 21.    

(Appendix 13A) Benz Company is considering the purchase of a machine that costs $200,000 and
has a useful life of 20 years. The company's required discount rate is 20%. If the machine's net
present value is $8,000, what must be the annual cash inflows associated with the machine,
rounded to the nearest whole dollar, do not round your intermediate calculations? (Ignore income
taxes in this problem.)

 $13,760.

 $42,714.

 $42,413.

 They are impossible to determine from the data given.

($200,000 + $8,000) value to use for PV Calculated using the PMT formula of Microsoft Excel.

https://ezto.mheducation.com/hm.tpx 13/98
12/16/21, 1:39 PM Assignment Print View

 
 22.    

One strength of the simple rate of return method is that it takes into account the time value of
money in computing the return on an investment project.

 True

 False

 
 23.    

(Appendix 13A) Vernon Company has been offered a seven-year contract to supply a part for the
military. After careful study, the company has estimated the following data relating to the contract:
 
Cost of equipment needed $400,000
Working capital needed $70,000
Annual cash receipts from the delivery of parts, $90,000
less cash operating costs
Salvage value of equipment at termination of the $10,000
contract

It is not expected that the contract would be extended beyond the initial contract period. The
company's discount rate is 14%. (Ignore income taxes in this problem.)

Required:

Use the net present value method to determine if the contract should be accepted. Round all
computations to the nearest dollar.

Description Years Amount 14% Factor Present Value


equipment 0 ($400,000) 1.0000 ($400,000)
Working capital 0 ($70,000) 1.0000 ($70,000)
Net annual cash        
inflow 1-7 $90,000 4.2883 $385,947
Salvage values,        
Equipment 7 $10,000 .3996 $3,996
Release of working        
capital 7 $70,000 .3996 $27,972
Net Present Value     ($52,085)  

The contract should not be accepted because the net present value is negative.

https://ezto.mheducation.com/hm.tpx 14/98
12/16/21, 1:39 PM Assignment Print View

 
 24.    

A company is considering creating a new division for a new product line the company is
considering offering. The division will be expected to generate revenues and expenses (including
depreciation) of $1,000,000 and $700,000 respectively. The division's start-up costs are estimated
at $2,000,000. Compute the proposed division's simple rate of return.

 25%

 12%

 15%

 20%

Simple rate of return = incremental operating revenues / initial investment = (incremental revenues
− incremental expenses including depreciation) / initial investment = ($1,000,000 − $700,000) /
$2,000,000 = 15%.

 
 25.    

A company is considering updating its obsolete plant and equipment. The new equipment would
be assumed to have a twenty-five year useful life. A discounted cash flow analysis of the costs and
benefits showed that the plant and equipment would have a net present value of ($600,000).
However, uncertainty exists as to the intangible benefits of upgrading the plant and equipment. The
company is subject to a cost of capital of 8%. How much would the plant and equipment's
intangible benefits have to amount to annually to make the investment worthwhile?

 $37,471

 $20,000

 $47,931

 $56,207

PVA (25 years, 8%): 10.675.


 
Additional cash flow per year required from intangible benefits = net present value to
be offset / present value factor = $600,000 / 10.675 = $56,207 annually (rounded).

https://ezto.mheducation.com/hm.tpx 15/98
12/16/21, 1:39 PM Assignment Print View

 
 26.    

RNXD Inc. has the opportunity to market a product for 5 years under a specialty contract. The
product will provide the company with net cash flows of $280,000. The investment calls for an
initial working capital investment $800,000. The investment also calls for the purchase of
equipment for $350,000. The machinery will have a salvage value of $40,000 at the end of the
contract. RNXD Inc. is subject to a 20% discount rate. The company is also subject to a tax rate of
45%. The net present value of this investment opportunity is:

 $74,720

 $14,170

 −$351,706

 $25,160

Net Present Value (rounded):


 
ITEM YEAR(S) PV
Purchase of equipment now $ −350,000.00
Working capital needed now $ −800,000.00
Cash flows from product line 1–5 $ 460,614.00
Salvage value of equipment 5 $ 16,080.00
Working capital released 5 $ 321,600.00
       
Net Present Value   $ −351,706.00
 

 
 27.    

The payback method of making capital budgeting decisions gives full consideration to the time
value of money.

 True

 False

https://ezto.mheducation.com/hm.tpx 16/98
12/16/21, 1:39 PM Assignment Print View

 
 28.    

The higher the profitability index, the more desirable the project.

 True

 False

 
 29.    

(Appendix 13B) If a firm acquires a depreciable asset on September 1, it can add only one-third of
the capital cost of the asset to the undepreciated capital cost (UCC) in the calculation of the capital
cost allowance (CCA) for the year.

 True

 False

 
 30.    

A local machine shop purchased a new milling machine yesterday for $92,874 which is expected to
generate annual cash savings of $12,000 for the next twelve years after which the machine will
have no salvage value. What is the machine's internal rate of return?

 7.5%

 10%

 12%

 18%

Present value factor of the internal rate of return = investment required / net annual cash inflow =
$92,874 / $12,000 = 7.7395. Looking this up under the 12-year column on PVA Table gives 7.5%.
This figure is exactly halfway between 7 and 8%, so you must interpolate.

https://ezto.mheducation.com/hm.tpx 17/98
12/16/21, 1:39 PM Assignment Print View

 
 31.    

RNXD Inc. has the opportunity to market a product for 5 years under a specialty contract. The
product will provide the company with net cash flows of $250,000. The investment calls for an
initial working capital investment $800,000. The investment also calls for the purchase of
equipment for $350,000. The machinery will have a salvage value of $30,000 at the end of the
contract. RNXD Inc. is subject to a 20% discount rate. The net present value of this investment
opportunity is:

 $74,720

 −$68,590

 −$39,330

 −$64,570

Net Present Value (rounded):


 
ITEM YEAR(S) PV
Purchase of equipment now $ −350,000.00
Working capital needed now $ −800,000.00
Cash flows from product line 1–5 $ 747,750
Salvage value of equipment 5 $ 12,060.00
Working capital released 5 $ 321,600.00
       
Net Present Value   $ −68,590.00
 

 
 32.    

(Appendix 13A) The present value concept considers both recovery of the original investment and
return on the original investment.

 True

 False

https://ezto.mheducation.com/hm.tpx 18/98
12/16/21, 1:39 PM Assignment Print View

 
 33.    

(Appendix 13A) Suppose an investment has cash inflows of R dollars at the end of each year for two
years. What will be the present value of these cash inflows using a 12% discount rate?

 Greater than under a 10% discount rate.

 Less than under a 10% discount rate.

 Equal to that under a 10% discount rate.

 Sometimes greater than under a 10% discount rate and sometimes less; it depends on R.

 
 34.    

(Appendix 13A) The Whitton Company uses a discount rate of 16%. The company has an opportunity
to buy a machine now for $18,000 that will yield cash inflows of $10,000 per year for each of the
next three years. The machine would have no salvage value. What is the net present value of this
machine, rounded to the nearest whole dollar, do not round your intermediate calculations? (Ignore
income taxes in this problem.)

 ($9,980)

 $4,459

 $12,000

 $22,460

Calculated using the NPV formula of Microsoft Excel.

https://ezto.mheducation.com/hm.tpx 19/98
12/16/21, 1:39 PM Assignment Print View

 
 35.    

GNRD Inc. has the opportunity to market a product for 5 years under a specialty contract. The
product will provide the company with net cash flows of $120,000. The investment calls for an initial
working capital investment $280,000. The investment also calls for the purchase of equipment for
$200,000. The machinery will have a salvage value of $60,000 at the end of the contract. GNRD
Inc. is subject to a 16% discount rate. The company is also subject to a tax rate of 38%. The net
present value of this investment opportunity is:

 $74,720

 $14,170

 −$74,574

 $25,160

Net Present Value (rounded):


ITEM YEAR(S) PV
Purchase of equipment now  -$200,000.00
Working capital needed now  -$280,000.00
Cash flows from product line 1–5   $243,585.60
Salvage value of equipment 5   $ 28,560.00
Working capital released 5   $133,280.00
     
Net Present Value    -$ 74,574.40

 
 36.    

(Appendix 13A) Sue Falls is the president of Sports, Inc. She is considering buying a new machine
that would cost $14,125. Sue has determined that the new machine promises an internal rate of
return of 12%, but Sue has misplaced the paper that gives the annual cost savings promised by the
new machine. She does remember that the machine has a projected life of ten years. Based on
these data, what are the annual cost savings? (Ignore income taxes in this problem.)

 It is impossible to determine from the data given.

 $1,412.50.

 $1,695.00.

 $2,500.00.

Calculated using the PMT formula of Microsoft Excel.


https://ezto.mheducation.com/hm.tpx 20/98
12/16/21, 1:39 PM Assignment Print View

 
 37.    

(Appendix 13B) Last year, the sales at Seidelman Company were $180,000 and were all cash sales.
The company's tax-deductible expenses were $20,000 and were all cash expenses. The tax rate
was 35%. What was the after-tax net cash inflow at Seidelman last year?

 $87,500

 $104,000

 $250,000

 $700,000

($180,000 - $20,000) * (1- 0.35) = $104,000.

 
 38.    

RNXD Inc. has the opportunity to market a product for 5 years under a specialty contract. The
product will provide the company with net cash flows of $325,000. The investment calls for an
initial working capital investment $350,000. The investment also calls for the purchase of
equipment for $700,000. The machinery will have a salvage value of $150,000 at the end of the
contract. RNXD Inc. is subject to a 24% discount rate. The net present value of this investment
opportunity is:

 $74,720

 $12,625

 −$39,330

 $25,160

Net Present Value (rounded):


 
ITEM YEAR(S) PV
Purchase of equipment now $ −700,000.00
Working capital needed now $ −350,000.00
Cash flows from product line 1–5 $ 892,125
Salvage value of equipment 5 $ 51,150.00
Working capital released 5 $ 119,350.00
       
Net Present Value   $ 12,625.00
 

https://ezto.mheducation.com/hm.tpx 21/98
12/16/21, 1:39 PM Assignment Print View

 
 39.    

(Appendix 13A) The following data are available on a proposed investment project:
 
Initial investment $142,500
Annual Cash Inflows $30,000
Life of the investment 8 years
Required Rate of Return 10%

Which of the following statements best describes the internal rate of return on the proposed
investment project? (Ignore income taxes in this problem.)

 It is between 11% and 12%.

 It is between 12% and 13%.

 It is between 13% and 14%.

 It is less than the required rate of return.

Calculated using the IRR formula of Microsoft Excel = 13.299%.

 
 40.    

(Appendix 13B) The reduction in taxes made possible by a capital cost allowance (CCA) tax shield
equals the amount of the CCA deduction multiplied by the tax rate.

 True

 False

https://ezto.mheducation.com/hm.tpx 22/98
12/16/21, 1:39 PM Assignment Print View

 
 41.    

(Appendix 13B) At the Bartholomew Company last year, all sales were for cash and all expenses
were paid in cash. The tax rate was 30%. If the after-tax net cash inflow from these operations last
year was $10,500, and if the total before-tax and tax-deductible cash expenses were $35,000, what
must have been the total before-tax cash sales?

 $45,000

 $50,000

 $60,000

 $65,000

10,500/(1 -.30) + 35,000 = $50,000.

 
 42.    

(Appendix 13A) White Company's required rate of return on capital budgeting projects is 12%. The
company is considering an investment opportunity that would yield a cash flow of $10,000 in five
years. What is the most that the company should be willing to invest in this project? (Ignore income
taxes in this problem.) (Round your PV factor to 5 decimal places and final answer to nearest whole
dollar amount.)

 $2,774

 $5,674

 $17,637

 $36,050

Calculated using the NPV formula of Microsoft Excel.

https://ezto.mheducation.com/hm.tpx 23/98
12/16/21, 1:39 PM Assignment Print View

 
 43.    

RNX Inc. purchased a machine for $50,000 which will save the company approximately $15,000 in
even cash flows per year. What is the machine's payback period?

 5 years

 3.33 years

 6.67 years

 6.25 years

Payback period (when the net annual cash inflow is the same every year) = investment required /
net annual cash inflow = $50,000 / $15000 = 3.33 years.

 
 44.    

(Appendix 13B) Suppose a machine that costs $80,000 has a useful life of eight years. Also
suppose that capital cost allowance (CCA) deduction on the machine is $10,000 in year 4. The tax
rate is 30%. What would be the tax savings from the CCA tax shield in year 4?

 $3,000 inflow.

 $3,200 outflow.

 $3,200 inflow.

 $4,800 inflow.

$10,000 * 0.3 = $3,000 inflow.

https://ezto.mheducation.com/hm.tpx 24/98
12/16/21, 1:39 PM Assignment Print View

 
 45.    

(Appendix 13A) Hotech Company has invested in a project that has an eight-year life. It is expected
that the annual cash inflow from the project will be $20,200. Assuming that the project has an
internal rate of return of 15%, how much was the initial investment in the project? (Ignore income
taxes in this problem.) (Round your PV factor to 5 decimal places and final answer to nearest whole
dollar amount.)

 $64,648

 $80,800

 $90,644

 $160,000

Calculated using the NPV formula of Microsoft Excel.

 
 46.    

A company is considering updating its obsolete plant and equipment. The new equipment would
be assumed to have a twenty-year useful life. A discounted cash flow analysis of the costs and
benefits showed that the plant and equipment would have a net present value of ($640,000).
However, uncertainty exists as to the intangible benefits of upgrading the plant and equipment. The
company is subject to a cost of capital of 15%. How much would the plant and equipment’s
intangible benefits have to amount to annually to make the investment worthwhile?

 $10,000

 $20,000

 $102,253

 $95,862

PVA (20 years, 15%): 6.259.


Additional cash flow per year required from intangible benefits = net present value to be offset /
present value factor = $640,000 / 6.259 = $102,253 annually (rounded).

https://ezto.mheducation.com/hm.tpx 25/98
12/16/21, 1:39 PM Assignment Print View

 
 47.    

If the net present value of a project is zero, based on a discount rate of 16%, which of the following
statements about the project's internal rate of return is correct?

 It is equal to 16%.

 It is less than 16%.

 It is greater than 16%.

 It cannot be determined from the information given.

 
 48.    

Which one of the following statements about the payback method of capital budgeting is correct?

 The payback method does NOT consider the time value of money.

 The payback method considers cash flows after the payback has been reached.

 The payback method uses discounted cash flow techniques.

 The payback method will lead to the same decision as other methods of capital
budgeting.

https://ezto.mheducation.com/hm.tpx 26/98
12/16/21, 1:39 PM Assignment Print View

 
 49.    

(Appendix 13A) Jim Bingham is considering starting a small catering business. He would need to
purchase a delivery van and equipment costing $145,000 to operate the business and another
$60,000 for inventories and other working capital needs. Rent for the building to be used by the
business will be $36,000 per year. Jim's marketing studies indicate that the annual cash inflow from
the business will amount to $140,000. In addition to the building rent, annual cash outflow for
operating costs will amount to $70,000. Jim wants to operate the catering business for only six
years. He estimates that the equipment could be sold at that time for 4% of its original cost. Jim
uses a 12% discount rate. (Ignore income taxes in this problem.)

Required:

Would you advise Jim to make this investment? Use Net Present Value and Profitability analysis to
support your decision.

Description Years Amount 12% Factor Present Value


Van & equipment 0 ($145,000) 1.0000 ($145,000)
Working capital 0 ($60,000) 1.0000 ($60,000)
Building rent 1-6 ($36,000) 4.1114 ($148,010)
Net annual cash        
inflow 1-6 $50,000 4.1114 $205,570
Salvage values,        
Equipment 6 $5,000 0.5066 $2,533
Release of working        
capital 6 $60,000 0.5066 $30,396
Net Present Value     (114,511)  

P.I. = (145,000 + 60,000 -114,511)/(145,000 + 60,000) = 0.44

It is advised that Jim do not make the investment because the net present value is negative, the PI
suggests that the investment would not break even because it is less than one and the earnings
sold for 4% of the original cost is relatively low and he therefore should consider alternative
investments.

https://ezto.mheducation.com/hm.tpx 27/98
12/16/21, 1:39 PM Assignment Print View

 
 50.    

(Appendix 13A) The following data concern an investment project:


 
Investment in Equipment $16,000
Net Annual cash inflows $3,600
Working capital required $4,500
Salvage value of the equipment $2,000
Life of the project $12 years
Discount rate $14%

The working capital will be released for use elsewhere at the conclusion of the project. (Ignore
income taxes in this problem.)

Required:

Calculate the project's net present value and the internal rate of return.

Item Year Amount 14%PV Factor Present Value


Investment Now ($16,000) 1.000 ($16,000)
Required
Net Annual Cash 1-12 3,600 5.660 20,376
Flows
Working capital        
Required Now (4,500) 1.000 (4,500)
Working capital        
Released 12 4,500 0.208 936
Salvage value        
Equipment 12 2,000 0.208 416
Net present value       $1,228

NPV = $1,226.19 and IRR = 15.24%. Calculated using the NPV and IRR formula of Microsoft Excel

https://ezto.mheducation.com/hm.tpx 28/98
12/16/21, 1:39 PM Assignment Print View

 
 51.    

The net present value method takes into account which of the following?
 
Cash Flow over Life of Project Time Value of Money
A) No Yes
B) No No
C) Yes No
D) Yes Yes

 Option A

 Option B

 Option C

 Option D

 
 52.    

Relevant cash outflows in the NPV and IRR methods include:

 Reduction in costs

 Initial investment

 Increased working capital needs

 Incremental operating costs

Reduction in costs is classified as a cash inflow.

https://ezto.mheducation.com/hm.tpx 29/98
12/16/21, 1:39 PM Assignment Print View

 
 53.    

(Appendix 13A) Parks Company is considering an investment proposal in which a working capital
investment of $10,000 would be required. The investment would provide cash inflows of $2,000
per year for six years. The working capital would be released for use elsewhere when the project is
completed. If the company's discount rate is 10%, what is the investment's net present value?
(Ignore income taxes in this problem.) (Do not round your intermediate calculations and round the
final answer to the nearest whole dollar.)

 $1,289

 ($1,289)

 $3,226

 $4,355

Calculated using the NPV formula of Microsoft Excel.

 
 54.    

(Appendix 13B) CCA is calculated by applying the prescribed rate to a declining balance called the
undepreciated capital cost (UCC).

 True

 False

https://ezto.mheducation.com/hm.tpx 30/98
12/16/21, 1:39 PM Assignment Print View

 
 55.    

(Appendix 13A) The following data pertain to an investment:


 
Cost of the Investment $20,000
Life of the Project 5 years
Annual Cost Savings $10,000
Estimated Salvage Value $1,000
Discount Rate 15%

What is the net present value of the proposed investment? (Ignore income taxes in this problem.)
(Do not round your intermediate calculations and round the final answer to the nearest whole
dollar.)

 ($3,430)

 $0

 $14,019

 $3,355

Calculated using the NPV formula of Microsoft Excel.

 
 56.    

A company with $800,000 in operating assets is considering the purchase of a machine that costs
$75,000 and which is expected to reduce operating costs by $20,000 each year. The payback
period for this machine in years is closest to which of the following? (Ignore income taxes in this
problem.)

 0.27 years.

 3.75 years.

 10.70 years.

 40.00 years.

75,000/20,000 = 3.75 years.

https://ezto.mheducation.com/hm.tpx 31/98
12/16/21, 1:39 PM Assignment Print View

 
 57.    

(Appendix 13A and 13B) A company anticipates a taxable cash receipt of $20,000 in year 3 of a
project. The company's tax rate is 30%, and its discount rate is 8%. What is the approximate present
value of this future cash flow? (Do not round your intermediate calculations and round the final
answer to the nearest whole dollar.)

 $4,763

 $6,000

 $11,114

 $14,000

20,000 * (1 -.30) = $14,000, then Calculated using the NPV formula of Microsoft Excel.

 
 58.    

(Appendix 13A) Horn Corporation is considering investing in a four-year project. Cash inflows from
the project are expected to be as follows: Year 1, $2,000; Year 2, $2,200; Year 3, $2,400; Year 4,
$2,600. If using a discount rate of 8%, the project has a positive net present value of $500, what
was the amount of the original investment? (Ignore income taxes in this problem.) (Do not round
your intermediate calculations and round the final answer to the nearest whole dollar.)

 $1,411

 $2,411

 $7,054

 $8,054

Calculated using the NPV formula of Microsoft Excel.

https://ezto.mheducation.com/hm.tpx 32/98
12/16/21, 1:39 PM Assignment Print View

 
 59.    

A local machine shop purchased a new milling machine yesterday for $170,280 which is expected
to generate annual cash savings of $20,000 for the next twenty years after which the machine will
have no salvage value. What is the machine’s internal rate of return?

 8%

 10%

 12.5%

 14%

Present value factor of the internal rate of return = investment required / net annual cash inflow =
$170,280 / $20,000 = 8.514. Looking this up under the 20-year column on PVA Table gives 10%.

 
 60.    

What is the capital budgeting method that divides a project's annual incremental net income by the
initial investment?

 The internal rate of return method.

 The simple (or accounting) rate of return method.

 The payback method.

 The net present value method.

https://ezto.mheducation.com/hm.tpx 33/98
12/16/21, 1:39 PM Assignment Print View

 
 61.    

(Appendix 13A) The following information concerns a proposed investment:


 
Investment Required $14,150
Annual Savings $2,500
Life of the Project 12 years

What is the internal rate of return? (Ignore income taxes in this problem.)

 5%

 10%

 12%

 14%

Calculated using the IRR formula of Microsoft Excel.

 
 62.    

(Appendix 13B) Consider a machine that costs $115,000 now and has a useful life of seven years.
This machine will require a major overhaul at the end of the fourth year that will cost X dollars. If the
tax rate is 40%, and if the after-tax cash outflow for this overhaul is $3,600, what is the amount of X
in dollars?

 $1,440

 $2,160

 $6,000

 $9,000

3,600/(1 -.40) = $6,000.

https://ezto.mheducation.com/hm.tpx 34/98
12/16/21, 1:39 PM Assignment Print View

 
 63.    

A local machine shop purchased a new milling machine yesterday for $108,186 which is expected
to generate annual cash savings of $19,500 for the next forty years after which the machine will
have no salvage value. What is the machine’s internal rate of return?

 7.5%

 10%

 12%

 18%

Present value factor of the internal rate of return = investment required / net annual cash inflow =
$108,186 / $19,500 = 5.548. Looking this up under the 40-year column on PVA Table gives 18%.

 
 64.    

Amani Company is studying a project that would have a ten-year life and would require a $480,000
investment in equipment that has no salvage value. The project would provide net income each
year as follows for the life of the project:
 
Sales   $500,000
Less cash variable expense   $280,000
Contribution margin   $220,000
Less fixed expenses:    
Fixed cash expenses $150,000  
Depreciation expenses $40,000 $190,000
Net income   $30,000

The company's required rate of return is 12%. What is the payback period for this project? (Ignore
income taxes in this problem and round your final answer to 2 decimal places.)

 6.86 years.

 3.00 years.

 4.28 years.

 9.00 years.

$480,000 / ($220,000 - $150,000) = 6.86 years.

https://ezto.mheducation.com/hm.tpx 35/98
12/16/21, 1:39 PM Assignment Print View

 
 65.    

(Appendix 13B) The capital cost allowance (CCA) tax shield of a Class 7 asset with a maximum 15%
CCA rate was $19,000 for Year 2. The income tax rate was 35%. What was the total CCA deduction
for the asset for Year 2? (Round your final answer to the nearest whole dollar).

 $12,000

 $20,000

 $30,00

 $54,286

$19,000/ 0.35 = $54,286.

 
 66.    

The payback period is the length of time it takes for an investment to recoup its own initial cost out
of the cash receipts it generates.

 True

 False

 
 67.    

(Appendix 13A) Stratford Company purchased a machine with an estimated useful life of seven
years. The machine will generate cash inflows of $90,000 each year over the next seven years. If
the machine has no salvage value at the end of seven years, and assuming the company's discount
rate is 10%, what is the purchase price of the machine if the net present value of the investment is
$170,000? (Ignore income taxes in this problem.) (Do not round your intermediate calculations and
round the final answer to the nearest whole dollar.)

 $170,000

 $221,950

 $268,158

 $608,157

Calculated using the NPV formula of Microsoft Excel.


https://ezto.mheducation.com/hm.tpx 36/98
12/16/21, 1:39 PM Assignment Print View

 
 68.    

The internal rate of return can be used for screening decisions.

 True

 False

 
 69.    

Information on four investment proposals is given below:


 
Proposal Investment Net Present Value
1 $50,000 $30,000
2 $60,000 $24,000
3 $30,000 $15,000
4 $45,000 $9,000

What are preference rankings of the four proposals according to the profitability index?

 3, 4, 1, 2.

 1, 2, 3, 4.

 1, 3, 2, 4.

 2, 1, 4, 3.

PI 1, 2, 3, 4 = (30 + 50)/50, (24 + 60)/60, (15 + 30)/30, (9 + 45)/45 = 1.6, 1.4, 1.5, 1.2 = 1, 3, 2, 4.

https://ezto.mheducation.com/hm.tpx 37/98
12/16/21, 1:39 PM Assignment Print View

 
 70.    

A company is considering creating a new division for a new product line the company is
considering offering. The division will be expected to generate revenues and expenses (including
depreciation) of $1,000,000 and $700,000 respectively. The division's start-up costs are estimated
at $750,000. Compute the proposed division's simple rate of return.

 25%

 12%

 40%

 20%

Simple rate of return = incremental operating revenues / initial investment = (incremental revenues
− incremental expenses including depreciation) / initial investment = ($1,000,000 − $700,000) /
$750,000 = 40%.

 
 71.    

(Appendix 13A and 13B) A company anticipates a capital cost allowance (CCA) deduction of
$30,000 in year 3 of a project. The company's tax rate is 30%, and its discount rate is 12%. What is
the approximate present value of the CCA tax shield resulting from this deduction? (Do not round
your intermediate calculations and round the final answer to the nearest whole dollar.)

 $6,406

 $9,000

 $14,947

 $21,000

30,000 * .30 = $9,000 then Calculated using the PV formula of Microsoft Excel.

https://ezto.mheducation.com/hm.tpx 38/98
12/16/21, 1:39 PM Assignment Print View

 
 72.    

(Appendix 13A and 13B) A company anticipates a capital cost allowance (CCA) deduction of
$20,200 in year 2 of a project. The company's tax rate is 45%, and its discount rate is 12%. What is
the approximate present value of the CCA tax shield resulting from this deduction? (Do not round
your intermediate calculations and round the final answer to the nearest whole dollar.)

 $7,246

 $6,000

 $11,161

 $14,000

$20,200 * 45% = $9,090 then Calculated using the PV formula of Microsoft Excel.

 
 73.    

A finance director must choose one among four of the following projects:
 
Project Net Present Value Initial Investment
A $ 140,000 $ 105,000
B $ 180,000 $ 120,000
C $ 200,000 $ 150,000
D $ 80,000 $ 64,000
 
Which of the above projects should the director choose last?

 A

 B

 C

 D

Project profitability index = present value of net cash inflows / investment required.
 
Project profitability index (A) = $140,000 / $105,000 = 1.33.
Project profitability index (B) = $180,000 / $120,000 = 1.50.
Project profitability index (C) = $200,000 / $150,000 = 1.33.
Project profitability index (D) = $80,000 / $64,000 = 1.25.
 
Project D has the lowest profitability index, so it should be the director’s last choice.

https://ezto.mheducation.com/hm.tpx 39/98
12/16/21, 1:39 PM Assignment Print View

 
 74.    

(Appendix 13A) An increase in the discount rate will result in an increase in the present value of a
given cash flow.

 True

 False

 
 75.    

A finance director must choose one among four of the following projects:
 
Project Net Present Value Initial Investment
A $ 210,000 $ 100,000
B $ 180,000 $ 90,000
C $ 200,000 $ 150,000
D $ 180,000 $ 64,000
 
Which of the above projects should the director choose first?

 A

 B

 C

 D

Project profitability index = present value of net cash inflows / investment required.
 
Project profitability index (A) = $210,000 / $100,000 = 2.10.
Project profitability index (B) = $180,000 / $90,000 = 2.00.
Project profitability index (C) = $200,000 / $150,000 = 1.33.
Project profitability index (D) = $180,000 / $64,000 = 2.81.
 
Project D has the highest profitability index, so it should be the director’s first choice.

https://ezto.mheducation.com/hm.tpx 40/98
12/16/21, 1:39 PM Assignment Print View

 
 76.    

(Appendix 13B) Which of the following items is NOT included in the formula for calculating the
present value of the capital cost allowance (CCA) tax shield?

 Amount of working capital to be released at the end of a project.

 The capital cost allowance rate.

 The firm's cost of capital.

 The firm's marginal income tax rate.

 
 77.    

If a project has a positive net value, the project should be rejected.

 True

 False

If a project has a positive net present value it means that it will make more than the required return
and thus will be accepted.

https://ezto.mheducation.com/hm.tpx 41/98
12/16/21, 1:39 PM Assignment Print View

 
 78.    

(Appendix 13A and 13B) Roy Company is trying to decide whether to invest in one of two projects: X
or Z. Associated data for each investment project follow:
 
  Project
  X Z
Cost of equipment $90,000 $140,000
Useful life 6 years 9 years
Annual net cash inflow $25,000 $30,000
Salvage value $8,000 $12,000

The equipment for each project is in Class 22 with a 30% maximum CCA rate. The income tax rate
is 30%. Roy's after-tax cost of capital is 12%.

Required:

a) Calculate the net present value of each project, and indicate which appears preferable in terms
of net present value.
b) Calculate the profitability index for each project, and indicate which project would be preferable
using this investment criterion.

a) The net present values of the projects are computed using the information below:
 
Project X Year Cash Flow Tax Effect After-tax Cash 10% Factor Present Value
Flow
Cost of Now $(90,000) 1.00 $(90,000) 1.000 $(90,000)
equipment
Net annual 1-6 $25,000 0.70 $17,500 4.111 71,943
cash inflow
Salvage value 6 $8,000 0.00 $8,000 0.507 $4,056
PV of CCA tax           $17,384
shield
Net present           $3,383
value

NPV of all cash flows = $(13,997.32). Add to that the PV of CCA tax savings $17,384 * and the NPV of
project X = $3,386.67. Calculated using the NPV formula of Microsoft Excel
* The present value (PV) of the CCA tax shield is calculated as follows:
First component (assuming zero salvage value):
 
PV = Cdt × (1 + 0.5k)
       (d + k)    (1 + k)
      = (90,000 × .30 × .30) × (1 + 0.5 × .12)
                (.30 + .12)                (1 + .12)
      = $19,2860 × 0.9464
      = $18,252

https://ezto.mheducation.com/hm.tpx 42/98
12/16/21, 1:39 PM Assignment Print View

Second component (correction for non-zero salvage value):


 
PV = (S × d ×t) × (1 + k)-n
            (d + k)

      = (8,000 × .30 × .30) × (1 + .12)-6


               (.30 + .12)
      = $1,714 × 0.5066
      = $868
Total present value = $18,252 - $868
                               = $17,384
 
Project Z Year Cash Flow Tax Effect After-tax Cash 10% Factor Present Value
Flow
Cost of Now $(140,000) 1.00 $(1400,000) 1.000 $(140,000)
equipment
Net annual 1-9 $30,000 0.70 $21,500 5.328 $111,888
cash inflow
Salvage value 9 $12,000 0.00 $12,000 0.361 $4,332
PV of CCA tax           $27,465
shield
Net present           $3,385
value

NPV of cash flows = $(23,779.43). Add to this the PV of CCA tax savings $27,465 * then the NPV of
project Z is $3685.57. Calculated using the NPV formula of Microsoft Excel
* The present value (PV) of the CCA tax shield is calculated as follows.
First component (assuming zero salvage value):
 
PV = Cdt × (1 + 0.5k)
        (d + k)   (1 + k)
      = (140,000 × .30 × .30) × (1 + 0.5 × 12)
                 (.30 + .12)                  (1 + .12)
      = $30,000 × 0.9464
      = $28,392

Second component (correction for non-zero salvage value):


 
PV = (S × d × t) × (1 + k)-n
      = (12,000 × .30 × .30) × (1 + .12)-9
                (.30 + .12)
      = $2,571 × 0.3606
      = $927
Total present value = $28,392 - $927
                               = $27,465

Decision: Project Z would seem to be preferable to Project X by a very small amount ($298.90 to
be exact.
b) PI = PV inflows/Investment.
For project X = (3,386.67 + 90,000)/90,000 = 1.0376
For project Z = (3,685.57 + 140,000)/140,000 = 1.0263
Decision: Project X is now preferred by 1% using the profitability index.

https://ezto.mheducation.com/hm.tpx 43/98
12/16/21, 1:39 PM Assignment Print View

 
 79.    

GNR Inc. purchased a machine for $40,000 which will save the company approximately $10,000 in
even cash flows per year. What is the machine's payback period?

 5 years

 3 years

 4 years

 3.6 years

Payback period (when the net annual cash inflow is the same every year) = investment required /
net annual cash inflow = $40,000 / $10,000 = 4 years.

 
 80.    

The net present value method assumes that cash flows from a project are immediately reinvested
at a rate of return equal to the discount rate.

 True

 False

 
 81.    

(Appendix 13B) Last year, a firm had taxable cash receipts of $1,000,000, and the tax rate was 40%.
What was the after-tax net cash inflow from these receipts?

 $240,000.

 $560,000.

 $600,000.

 $800,000.

$1,000,000 * (1-0.4) = $600,000.

https://ezto.mheducation.com/hm.tpx 44/98
12/16/21, 1:39 PM Assignment Print View

 
 82.    

Which of the following is a weakness of the internal rate of return method for screening investment
projects?

 It does NOT consider the time value of money.

 It implicitly assumes that the company is able to reinvest cash flows from the project at the
company's discount rate.

 It implicitly assumes that the company is able to reinvest cash flows from the project at the
internal rate of return.

 It does NOT take into account all of the cash flows from a project.

 
 83.    

What is the present value of a 10-year, $6,000 after-tax annual cash flow stream at 10%? Assume
the cash flows occur at the start of each period.

 $34,554

 $61,450

 $80,000

 $69,108

PVA factor (9 years, 10%) = 5.759.


$6,000 × 5.759 = $34,554.

https://ezto.mheducation.com/hm.tpx 45/98
12/16/21, 1:39 PM Assignment Print View

 
 84.    

What is the present value of a 10-year, $18,000 after-tax annual cash flow stream at 10%? Assume
the cash flows occur at the end of each period and that the first cash flows occur 3 years from now.

 $50,364

 $75,547

 $46,149

 $88,000

PVA factor (10 years, 10%) = 6.145.


$18,000 × 6.145 = $110,610.
 
PV factor (3 years, 10%) = 0.751.
$110,610 × 0.683 = $75,547.

 
 85.    

RNXD Inc. has the opportunity to market a product for 5 years under a specialty contract. The
product will provide the company with net cash flows of $200,000. The investment calls for an
initial working capital investment $300,000. The investment also calls for the purchase of
equipment for $500,000. The machinery will have a salvage value of $80,000 at the end of the
contract. RNXD Inc. is subject to a 22% discount rate. The net present value of this investment
opportunity is:

 $74,720

 $14,170

 −$39,330

 −$86,600

Net Present Value (rounded):


 
ITEM YEAR(S) PV
Purchase of equipment now $ −500,000.00
Working capital needed now $ −300,000.00
Cash flows from product line 1–5 $ 572,800.00
Salvage value of equipment 5 $ 29,600.00
Working capital released 5 $ 111,000.00
       
Net Present Value   $ −86,600.00
 

https://ezto.mheducation.com/hm.tpx 46/98
12/16/21, 1:39 PM Assignment Print View

 
 86.    

(Appendix 13B) By what amount does a capital cost allowance (CCA) deduction reduce income
taxes?

 One minus the tax rate multiplied by the amount of the CCA deduction.

 The tax rate multiplied by the amount of the CCA deduction.

 The amount of the CCA deduction.

 The amount of the CCA deduction divided by one minus the tax rate.

 
 87.    

What does the payback method measure?

 How quickly investment dollars may be recovered.

 The cash flow from an investment.

 The economic life of an investment.

 The profitability of an investment.

 
 88.    

(Appendix 13A and 13B) A company needs an increase in working capital of $70,000 in a project
that will last three years. The company's tax rate is 30%, and its discount rate is 8%. What is the
approximate present value of the working capital to be released at the end of the project? (Do not
round your intermediate calculations and round the final answer to the nearest whole dollar.)

 $21,000

 $38,898

 $49,000

 $55,568

Calculated using the PV formula of Microsoft Excel.

https://ezto.mheducation.com/hm.tpx 47/98
12/16/21, 1:39 PM Assignment Print View

 
 89.    

GNRD Inc. has the opportunity to market a product for 5 years under a specialty contract. The
product will provide the company with net cash flows of $280,000. The investment calls for an
initial working capital investment $800,000. The investment also calls for the purchase of
equipment for $350,000. The machinery will have a salvage value of $40,000 at the end of the
contract. GNRD Inc. is subject to a 20% discount rate. The company is also subject to a tax rate of
45%. The net present value of this investment opportunity is:

 $74,720

 $14,170

 −$351,706

 $25,160

Net Present Value (rounded):


ITEM YEAR(S) PV
Purchase of equipment now  -$350,000.00
Working capital needed now  -$800,000.00
Cash flows from product line 1–5   $460,614.00
Salvage value of equipment 5   $ 16,080.00
Working capital released 5   $321,600.00
     
Net Present Value    -$351,706.00

https://ezto.mheducation.com/hm.tpx 48/98
12/16/21, 1:39 PM Assignment Print View

 
 90.    

A finance director must choose one among four of the following projects:
 
Project Net Present Value Initial Investment
A $ 200,000 $ 100,000
B $ 160,000 $ 90,000
C $ 200,000 $ 150,000
D $ 180,000 $ 60,000
 
Which of the above projects should the director choose first?

 A

 B

 C

 D

Project profitability index = present value of net cash inflows / investment required.
 
Project profitability index (A) = $200,000 / $100,000 = 2.00.
Project profitability index (B) = $160,000 / $90,000 = 1.78.
Project profitability index (C) = $200,000 / $150,000 = 1.33.
Project profitability index (D) = $180,000 / $60,000 = 3.00.
 
Project D has the highest profitability index, so it should be the director’s first choice.

https://ezto.mheducation.com/hm.tpx 49/98
12/16/21, 1:39 PM Assignment Print View

 
 91.    

What is the present value of a 10-year, $15,000 after-tax annual cash flow stream at 10%? Assume
the cash flows occur at the end of each period and that the first cash flows occur 4 years from now.

 $41,970

 $62,956

 $80,000

 $88,000

PVA factor (10 years, 10%) = 6.145.


$15,000 × 6.145 = $92,175.
 
PV factor (4 years, 10%) = 0.683.
$92,175 × 0.683 = $62,956.

 
 92.    

The internal rate of return for a project is the discount rate that makes the net present value of the
project equal to zero.

 True

 False

https://ezto.mheducation.com/hm.tpx 50/98
12/16/21, 1:39 PM Assignment Print View

 
 93.    

What is the present value of a 10-year, $15,000 after-tax annual cash flow stream at 10%? Assume
the cash flows occur at the end of each period.

 $50,000

 $92,175

 $80,000

 $88,000

PVA factor (10 years, 10%) = 6.145.


$15,000 × 6.145 = $92,175.

 
 94.    

(Appendix 13B) If a company operates at a profit, the after-tax cost of a tax-deductible cash expense
is determined by multiplying the cash expense by one minus the tax rate.

 True

 False

 
 95.    

Preference decisions relate to selecting from among several acceptable alternatives.

 True

 False

https://ezto.mheducation.com/hm.tpx 51/98
12/16/21, 1:39 PM Assignment Print View

 
 96.    

(Appendix 13A and 13B) A company anticipates a taxable cash receipt of $90,000 in year 5 of a
project. The company's tax rate is 35%, and its discount rate is 15%. What is the approximate
present value of this future cash flow? (Do not round your intermediate calculations and round the
final answer to the nearest whole dollar.)

 $10,125

 $15,000

 $29,085

 $35,000

$90,000 * (1-0.35) = $58,500 then Calculated using the PV formula of Microsoft Excel.

 
 97.    

The preference rule for ranking projects by the profitability index is the following: the higher the
profitability index, the more desirable the project.

 True

 False

https://ezto.mheducation.com/hm.tpx 52/98
12/16/21, 1:39 PM Assignment Print View

 
 98.    

RNXD Inc. has the opportunity to market a product for 5 years under a specialty contract. The
product will provide the company with net cash flows of $120,000. The investment calls for an initial
working capital investment $280,000. The investment also calls for the purchase of equipment for
$200,000. The machinery will have a salvage value of $30,000 at the end of the contract. RNXD
Inc. is subject to a 16% discount rate. The company is also subject to a tax rate of 33.33%. The net
present value of this investment opportunity is:

 $74,720

 −$43,144

 −$70,519

 $25,160

Net Present Value (rounded):


 
ITEM YEAR(S) PV
Purchase of equipment now $ −200,000.00
Working capital needed now $ −280,000.00
Cash flows from product line 1–5 $ 261,921.31
Salvage value of equipment 5 $ 14,280.00
Working capital released 5 $ 133,280.00
       
Net Present Value   $ −70,518.69
 

https://ezto.mheducation.com/hm.tpx 53/98
12/16/21, 1:39 PM Assignment Print View

 
 99.    

A finance director must choose one among four of the following projects:
 
Project Net Present Value Initial Investment
A $ 160,000 $ 105,000
B $ 180,000 $ 120,000
C $ 200,000 $ 150,000
D $ 80,000 $ 64,000
 
Which of the above projects should the director choose last?

 A

 B

 C

 D

Project profitability index = present value of net cash inflows / investment required.
 
Project profitability index (A) = $160,000 / $105,000 = 1.52.
Project profitability index (B) = $180,000 / $120,000 = 1.50.
Project profitability index (C) = $200,000 / $150,000 = 1.33.
Project profitability index (D) = $80,000 / $64,000 = 1.25.
 
Project D has the lowest profitability index, so it should be the director’s last choice.

 
 100.    

ABC Inc. needs a new machine. The company is considering two options:
Machine 1: purchase a machine for $50,000 which will save the company approximately $15,000 in
operating costs per year.
Machine 2: purchase a machine for $40,000 which will save the company approximately $13,000 in
operating costs per year.
Which machine should be purchased according to the payback method.

 Machine 1

 Machine 2

https://ezto.mheducation.com/hm.tpx 54/98
12/16/21, 1:39 PM Assignment Print View

 
 101.    

(Appendix 13B) The working capital required at the start of an investment is a tax deductible cash
outflow.

 True

 False

 
 102.    

(Appendix 13A) The following data pertain to an investment proposal:


 
Present investment required $14,000
Annual cost savings $2,500
Projected life of the investment 8 years
Projected Salvage Value $-0-
Required Rate of Return 6%

The internal rate of return on this investment proposal is closest to which of the following? (Ignore
income taxes in this problem.)

 6.7%

 7.3%

 8.7%

 9.3%

Calculated using the IRR formula of Microsoft Excel.

https://ezto.mheducation.com/hm.tpx 55/98
12/16/21, 1:39 PM Assignment Print View

 
 103.    

GNRD Inc. has the opportunity to market a product for 5 years under a specialty contract. The
product will provide the company with net cash flows of $40,000. The investment calls for an initial
working capital investment $280,000. The investment also calls for the purchase of equipment for
$200,000. The machinery will have a salvage value of $60,000 at the end of the contract. GNRD
Inc. is subject to a 16% discount rate. The net present value of this investment opportunity is:

 $74,720

 $14,170

 −$187,200

 $25,160

Net Present Value (rounded):


 
 
ITEM YEAR(S) PV
Purchase of equipment now $ −200,000.00
Working capital needed now $ −280,000.00
Cash flows from product line 1–5 $ 130,960.00
Salvage value of equipment 5 $ 28,560.00
Working capital released 5 $ 133,280.00
       
Net Present Value   $ −187,200.00
 

 
 104.    

(Appendix 13A and 13B) A company anticipates a tax-deductible cash expense of $10,000 in year 2
of a project. The company's tax rate is 30%, and its discount rate is 8%. What is the approximate
present value of this future cash outflow? Do not round your intermediate calculations and round
the final answer to the nearest whole dollar.)

 $7,000

 $6001

 $3,000

 $2,572

10,000 * (1 -.30) = $7,000 then Calculated using the PV formula of Microsoft Excel.

https://ezto.mheducation.com/hm.tpx 56/98
12/16/21, 1:39 PM Assignment Print View

 
 105.    

(Appendix 13A) The following data pertain to an investment proposal:


 
Present Investment Required $27,130
Annual Cost Savings $5,000
Projected life the investment 10 years
Projected Salvage Value $-0-

What would be the internal rate of return? (Ignore income taxes in this problem.)

 5.426%

 13.0%

 54.26%

 542.6%

Calculated using the IRR formula of Microsoft Excel.

 
 106.    

GNR Inc. purchased a machine for $36,000 which will save the company approximately $6,000 in
even cash flows per year. What is the machine's payback period?

 4 years

 6 years

 3.33 years

 3.6 years

Payback period (when the net annual cash inflow is the same every year) = investment required /
net annual cash inflow = $36,000 / $6,000 = 6 years.

https://ezto.mheducation.com/hm.tpx 57/98
12/16/21, 1:39 PM Assignment Print View

 
 107.    

Why are the net present value and internal rate of return methods of capital budgeting superior to
the payback method?

 Because they are easier to implement.

 Because they consider the time value of money.

 Because they require less input.

 Because they reflect the effects of depreciation and income taxes.

 
 108.    

(Appendix 13B) At what amount should the capital cost allowance (CCA) tax shield be included in
the calculation of the net present value of an investment project?

 The amount of the CCA with no adjustment for taxes.

 The amount of the CCA multiplied by one minus the tax rate.

 The amount of the CCA multiplied by the tax rate.

 Zero, since the amount of CCA is not relevant to the calculation of net present value.

 
 109.    

(Appendix 13A) Sam Weller is thinking of investing $70,000 to start a bookstore. Sam plans to earn
$15,000 cash from the business at the end of each year for the next five years. At the end of the
fifth year, Sam plans to sell the business for $110,000 cash. At a 12% discount rate, what is the net
present value of the investment? (Ignore income taxes in this problem.) (Do not round your
intermediate calculations and round the final answer to the nearest whole dollar.)

 $46,489

 $54,075

 $62,370

 $70,000

Calculated using the NPV formula of Microsoft Excel.

https://ezto.mheducation.com/hm.tpx 58/98
12/16/21, 1:39 PM Assignment Print View

 
 110.    

RNXD Inc. has the opportunity to market a product for 5 years under a specialty contract. The
product will provide the company with net cash flows of $40,000. The investment calls for an initial
working capital investment $280,000. The investment also calls for the purchase of equipment for
$200,000. The machinery will have a salvage value of $45,000 at the end of the contract. RNXD
Inc. is subject to a 16% discount rate. The net present value of this investment opportunity is:

 $74,720

 $14,170

 −$187,200

 −$194,340

Net Present Value (rounded):


 
ITEM YEAR(S) PV
Purchase of equipment now $ −200,000.00
Working capital needed now $ −280,000.00
Cash flows from product line 1–5 $ 130,960.00
Salvage value of equipment 5 $ 21,420.00
Working capital released 5 $ 133,280.00
       
Net Present Value   $ −194,340.00
 

 
 111.    

(Appendix 13A and 13B) A company needs an increase in working capital of $20,000 in a project
that will last four years. The company's tax rate is 30%, and its discount rate is 10%. What is the
approximate present value of the working capital to be released at the end of the project? (Do not
round your intermediate calculations and round the final answer to the nearest whole dollar.)

 $6,000

 $9,562

 $13,660

 $14,000

Calculated using the PV formula of Microsoft Excel.

https://ezto.mheducation.com/hm.tpx 59/98
12/16/21, 1:39 PM Assignment Print View

 
 112.    

Projects with shorter payback periods are always more profitable than projects with longer payback
periods.

 True

 False

 
 113.    

RNX Inc. purchased a machine for $40,000 which will save the company approximately $16,000 in
even cash flows per year. What is the machine's payback period?

 5 years

 3 years

 2.5 years

 3.6 years

Payback period (when the net annual cash inflow is the same every year) = investment required /
net annual cash inflow = $40,000 / $16,000 = 2.5 years.

https://ezto.mheducation.com/hm.tpx 60/98
12/16/21, 1:39 PM Assignment Print View

 
 114.    

A company is considering updating its obsolete plant and equipment. The new equipment would
be assumed to have a fifteen-year useful life. A discounted cash flow analysis of the costs and
benefits showed that the plant and equipment would have a net present value of ($340,000).
However, uncertainty exists as to the intangible benefits of upgrading the plant and equipment. The
company is subject to a cost of capital of 14%. How much would the plant and equipment’s
intangible benefits have to amount to annually to make the investment worthwhile?

 $10,000

 $20,000

 $30,000

 $55,357

PVA (15 years, 14%): 6.142.


Additional cash flow per year required from intangible benefits = net present value to be offset /
present value factor = $340,000 / 6.142 = $55,357 annually (rounded).

 
 115.    

A local machine shop purchased a new milling machine yesterday for $81,611.25 which is expected
to generate annual cash savings of $15,000 for the next eight years after which the machine will
have no salvage value. What is the machine's internal rate of return?

 8%

 10%

 12%

 9.46%

Present value factor of the internal rate of return = investment required / net annual cash inflow =
$81,611.25 / $15,000 = 5.44075. Looking this up under the 8-year column on PVA Table gives 16.5%.
This figure is exactly halfway between 16 and 17%, so you must interpolate.

https://ezto.mheducation.com/hm.tpx 61/98
12/16/21, 1:39 PM Assignment Print View

 
 116.    

(Appendix 13A and 13B) Suppose that your company has ample taxable income to take advantage
of capital cost allowance (CCA) deductions and has the option to use either accelerated or straight-
line method to calculate the CCA deductions.

Required:

Which method would you prefer? Why?

The accelerated method of capital cost allowance (CCA) would be preferable. It will provide the
bulk of the CCA tax shields early in the life of the asset, whereas the straight-line method will
spread the tax shields out evenly over the entire life of the asset. It is generally better to have the
tax reductions early in the life of an asset than to have them later because of the time value of
money.

 
 117.    

A company is considering buying a new machine for $40,000. The machine is expected to save the
company $16,000 for each of the next 5 years. The cost of the machine will be depreciated evenly
over the next 5 years, with no salvage value. Compute the machine's simple rate of return. Ignore
income taxes.

 30%

 10%

 15%

 20%

Simple rate of return = incremental operating revenues / initial investment = (incremental revenues
− incremental expenses including depreciation) / initial investment = ($16,000 − ($40,000 / 5)) /
$40,000 = 20%.

https://ezto.mheducation.com/hm.tpx 62/98
12/16/21, 1:39 PM Assignment Print View

 
 118.  
 

A local machine shop purchased a new milling machine yesterday for $115,047 which is expected to
generate annual cash savings of $9,000 for the next twenty-five years after which the machine will
have no salvage value. What is the machine’s internal rate of return?

 8%

 10%

 12%

 6%

Present value factor of the internal rate of return = investment required / net annual cash inflow =
$115,047 / $9,000 = 12.783. Looking this up under the 25-year column on PVA Table gives 6%.

 
 119.    

(Appendix 13A) A piece of equipment has a cost of $20,000. The equipment will provide cost
savings of $3,500 each year for ten years, after which time it will have a salvage value of $2,500. If
the company's discount rate is 12%, what is the equipment's net present value? (Ignore income
taxes in this problem.) (Do not round your intermediate calculations and round the final answer to
the nearest whole dollar.)

 ($224)

 $581

 $546

 $17,500

Calculated using the NPV formula of Microsoft Excel.

https://ezto.mheducation.com/hm.tpx 63/98
12/16/21, 1:39 PM Assignment Print View

 
 120.    

The net present value method of capital budgeting assumes that cash flows are reinvested at what
rate?

 The internal rate of return on the project.

 The rate of return on the company's debt.

 The discount rate used in the analysis.

 A zero rate of return.

 
 121.    

(Appendix 13A) The present value of a given sum to be received in five years will be exactly twice
as great as the present value of an equal sum to be received in ten years.

 True

 False

 
 122.    

The Jason Company is considering the purchase of a machine that will increase revenues by
$32,000 each year. Cash outflows for operating this machine will be $6,000 each year. The cost of
the machine is $65,000. It is expected to have a useful life of five years with no salvage value. For
this machine, what is the simple rate of return? (Ignore income taxes in this problem.)

 9.2%

 20.0%

 40.0%

 49.2%

(32,000 - 6,000 - 65,000/5)/65,000 = 20%.

https://ezto.mheducation.com/hm.tpx 64/98
12/16/21, 1:39 PM Assignment Print View

 
 123.    

The Amani Company is planning a $200,000 equipment investment that has an estimated eight-
year life with no estimated salvage value. The company has projected the following annual cash
flows for the investment.
 
Year Cash Inflows
1 $120,000
2 $70,000
3 $60,000
4 $60,000
5 $60,000
Total $370,000

Assuming that the cash inflows occur evenly over the year, what is the payback period for the
investment? (Ignore income taxes in this problem.)

 0.75 years.

 1.67 years.

 2.17 years.

 4.91 years.

$200,000 - $120,000 - $70,000 = $10,000 remaining after year 2;


Payback = 2 + $10,000 / $60,000 = 2.17 years.

 
 124.    

If net present value of a project is negative, it should be rejected.

 True

 False

If a project has a negative net present value it means the project will make less than the required
return.

https://ezto.mheducation.com/hm.tpx 65/98
12/16/21, 1:39 PM Assignment Print View

 
 125.    

(Appendix 13A) Bradley Company's required rate of return is 10%. The company has an opportunity
to be the exclusive distributor of a very popular consumer item. No new equipment would be
needed, but the company would have to use one-fourth of the space in a warehouse it owns. The
warehouse cost $160,400 new. The warehouse is currently half-empty, and there are no other plans
to use the empty space. In addition, the company would have to invest $94,000 in working capital
to carry inventories and accounts receivable for the new product line. The company would have the
distributorship for only five years. The distributorship would generate a $19,000 net annual cash
inflow. (Ignore income taxes in this problem.)

Required:

What is the net present value of the project at a discount rate of 10%? Should the project be
accepted?

  Year Amount 10%Factor Present Value


Investment Now $(94,000) 1.0000 $(94,000)
Required
Annual Cash Flows 1-5 19,000 3.7908 72,025
Working capital 5 94,000 0.6209 58,365
released
Net present value       $35,390

Yes, the distributorship should be accepted because the project has a positive net present value.

 
 126.    

(Appendix 13B) Capital cost allowance (CCA) deductions shield revenues from taxation and thereby
lower the amount of taxes that a company must pay.

 True

 False

https://ezto.mheducation.com/hm.tpx 66/98
12/16/21, 1:39 PM Assignment Print View

Sunshade Company has gathered the following data on a proposed investment project:
 
Investment Required in Equipment $480,000
Annual Cash Inflows $80,000
Salvage Value $-0-
Life of the Investment 8 years
Discount Rate 5%

(Ignore income taxes in this problem.)

 
 127.    

The payback period for the investment is closest to which of the following?

 0.2 years.

 6.0 years.

 3.0 years.

 5.0 years.

$480,000 / $80,000 = 6 years.

 
 128.    

The simple rate of return on the investment is closest to which of the following? (Round your
percentage answer to the nearest number.)

 5%

 4%

 15%

 20%

($80,000 - $480,000/8)/ $480,000 = 4.17% = 4% (rounded).

https://ezto.mheducation.com/hm.tpx 67/98
12/16/21, 1:39 PM Assignment Print View

 
 129.    

(Appendix 13A) The net present value on this investment is closest to which of the following? (Do
not round your intermediate calculations and round your final answer to nearest whole dollar.)

 $76,750

 $80,000

 $37,057

 $400,000

Calculated using the NPV formula of Microsoft Excel.

 
 130.    

(Appendix 13A) The internal rate of return on the investment is closest to which of the following?
(Round your percentage answer to the nearest number.)

 10%

 13%

 7%

 17%

Calculated using the IRR formula of Microsoft Excel.

Bugle's Bagel Bakery is investigating the purchase of a new bagel-making machine. This machine
would provide an annual operating cost savings of $3,650 for each of the next four years. In
addition, this new machine would allow the production of one new type of bagel that would result in
selling 1,500 dozen more bagels each year. The company earns a contribution margin of $0.90 on
each dozen bagels sold. The purchase price of this machine is $13,450, and it will have a four-year
useful life. Bugle's discount rate is 14%. (Ignore income taxes in this problem.)

https://ezto.mheducation.com/hm.tpx 68/98
12/16/21, 1:39 PM Assignment Print View

 
 131.    

What is the total annual cash inflow from this machine for capital budgeting purposes?

 $3,650

 $4,750

 $5,000

 $5,150

1,500 * .90 + 3,650 = $5,000.

 
 132.    

(Appendix 13A) The internal rate of return for this investment is closest to which of the following?

 14%

 16%

 18%

 20%

Calculated using the IRR formula of Microsoft Excel.

 
 133.    

(Appendix 13A) The net present value of this investment is closest to which of the following? (Do not
round your intermediate calculations.)

 $1,119

 $6,550

 $13,450

 $20,000

Calculated using the NPV formula of Microsoft Excel.

https://ezto.mheducation.com/hm.tpx 69/98
12/16/21, 1:39 PM Assignment Print View

Treads Corporation is considering the replacement of an old machine that is currently being used.
The old machine is fully depreciated but can be used by the corporation for five more years. If
Treads decides to replace the old machine, Picco Company has offered to purchase the old machine
for $60,000. The old machine would have no salvage value in five years.
The new machine would be acquired from Hillcrest Industries for $1,000,000 in cash. The new
machine has an expected useful life of five years with no salvage value. Due to the increased
efficiency of the new machine, estimated annual cash savings of $300,000 would be generated.
Treads Corporation uses a discount rate of 12%. (Ignore income taxes in this problem.)

 
 134.    

(Appendix 13A) The net present value of the project is closest to which of the following? (Do not
round your intermediate calculations.)

 $136,400

 $141,500

 $171,000

 $560,000

Calculated using the NPV formula of Microsoft Excel.

 
 135.    

(Appendix 13A) The internal rate of return of the project is closest to which of the following?

 12%

 16%

 18%

 20%

Calculated using the IRR formula of Microsoft Excel.

https://ezto.mheducation.com/hm.tpx 70/98
12/16/21, 1:39 PM Assignment Print View

Switch Manufacturing has gathered the following data on a proposed investment project:
 
Investment in Depreciable Equipment $200,200
Annual Net Cash Flows $70,000
Life of the Equipment 6 years
Salvage Value -0-
Discount Rate 12%

The company uses straight-line depreciation on all equipment. (Ignore income taxes in this problem.)

 
 136.    

What would be the payback period for the investment?

 0.25 years.

 2.41 years.

 2.86 years.

 10.00 years.

$200,200 / $70,000 = 2.86 years.

 
 137.    

What would be the simple rate of return on the investment? (Round your percentage answer to the
nearest number.)

 10%

 15%

 18%

 35%

($70,000 - $200,200/ 6) / $200,200 = 18.3% = 18% (rounded).

https://ezto.mheducation.com/hm.tpx 71/98
12/16/21, 1:39 PM Assignment Print View

 
 138.    

(Appendix 13A) What would be the net present value of this investment? (Do not round your
intermediate calculations and round your final answer to the nearest whole dollar.)

 ($14,350)

 $77,200

 $87,598

 $200,000

Calculated using the NPV Table.

Apex Corp., is planning to buy a production machine costing $490,000. This machine's expected
useful life is five years, with no residual value. Apex uses a discount rate of 15% and has calculated
the following data pertaining to the purchase and operation of this machine:
 
Year Estimated Annual Net Cash Inflow
1 $120,000
2 $100,000
3 $80,000
4 $80,000
5 $80,000

(Ignore income taxes in this problem.)

 
 139.    

What is the payback period for this investment?

 2.50 years.

 2.75 years.

 5.38 years.

 5.00 years.

$490,000 - $120,000 - $100,000 = $270,000;


Payback = 2 + $270,000 /$80,000 = 5.38 years.
https://ezto.mheducation.com/hm.tpx 72/98
12/16/21, 1:39 PM Assignment Print View

 
 140.    

(Appendix 13A) The net present value of this investment is closest to which of the following? (Do not
round your intermediate calculations.)

 $(171,922)

 $81,025

 $50,000

 $80,000

Calculated using the NPV formula of Microsoft Excel.

The Finney Company is reviewing the possibility of remodelling one of its showrooms and buying
some new equipment to improve sales operations. The remodelling would cost $120,000 now, and
the useful life of the project is ten years. Additional working capital needed immediately for this
project would be $30,000; the working capital would be released for use elsewhere at the end of
the ten-year period. The equipment and other materials used in the project would have a salvage
value of $10,000 in ten years. Finney's discount rate is 16%. (Ignore income taxes in this problem.)

 
 141.    

What would be the immediate cash outflow required for this project?

 ($150,000)

 ($130,000)

 ($120,000)

 ($90,000)

-120,000 - 30,000 = ($150,000).

https://ezto.mheducation.com/hm.tpx 73/98
12/16/21, 1:39 PM Assignment Print View

 
 142.    

(Appendix 13A) What would the annual net cash inflows from this project have to be in order to
justify investing in remodelling? (Do not round your intermediate calculations and round your final
answer to the nearest whole number.)

 $14,495

 $16,147

 $29,159

 $35,842

Calculated using the NPV and PMT formula of Microsoft Excel. NPV = ($140,932.66) then PMT =
$29,159.

The Sawyer Company has $80,000 to invest and is considering two different projects: X and Y. The
following data are available on the projects:
 
  Project X Project Y
Cost of equipment needed now $80,000 ---
Working capital requirement --- $80,000
Annual cash operating inflows $23,000 $18,000
Salvage value in five years $6,000 ---

Both projects will have a useful life of five years; at the end of five years, the working capital will be
released for use elsewhere. Sawyer's discount rate is 12%. (Ignore income taxes in this problem.)

 
 143.    

(Appendix 13A) What is the net present value of project X? (Do not round your intermediate
calculations and round your final answer to the nearest whole number.)

 ($11,708)

 $2,910

 $5,283

 $6,314

Calculated using the NPV formula of Microsoft Excel.

https://ezto.mheducation.com/hm.tpx 74/98
12/16/21, 1:39 PM Assignment Print View

 
 144.    

(Appendix 13A) The net present value of project Y is closest to which of the following? (Do not
round your intermediate calculations.)

 ($11,708)

 $11,708

 $20,066

 $30,280

Calculated using the NPV formula of Microsoft Excel.

The Becker Company is interested in buying a piece of equipment. The following data have been
assembled concerning this equipment:
 
Cost of Required Equipment $250,000
Working capital required $100,000
Annual operating cash inflows $80,000
Cash repair at End of Four years $40,000
Salvage Value at End of Six Years $90,000

This equipment is expected to have a useful life of six years. At the end of the sixth year, the working
capital would be released for use elsewhere. The company's discount rate is 10%. (Ignore income
taxes in this problem.)

 
 145.    

(Appendix 13A) The present value of all future annual operating cash inflows is closest to which of
the following? (Do not round your intermediate calculations.)

 $278,700

 $348,421

 $452,301

 $480,000

Calculated using the PV formula of Microsoft Excel.

https://ezto.mheducation.com/hm.tpx 75/98
12/16/21, 1:39 PM Assignment Print View

 
 146.    

(Appendix 13A) The net present value of this investment is closest to which of the following? Do not
round your intermediate calculations.)

 $78,350

 $21,904

 $27,548

 $54,640

Calculated using the NPV formula of Microsoft Excel.

 
 147.    

(Appendix 13A) The internal rate of return for this investment is closest to which of the following?

 10%

 16%

 12%

 13%

Calculated using the IRR formula of Microsoft Excel.

Auto Company is considering rebuilding and selling used alternators for automobiles. The company
estimates that the net operating cash flows (sales minus cash operating expenses) arising from the
rebuilding and sale of the used alternators would be as follows (numbers in parentheses indicate an
outflow):
 
Year 1 - 10 $90,000
Year 11 ($50,000)
Year 12 $190,000

In addition to the above net operating cash flows, Auto Company would purchase production
equipment costing $200,000 now to use in the rebuilding of the alternators. The equipment would
have a 12 -year life and a $18,000 salvage value. The company's discount rate is 15%. (Ignore income
taxes in this problem.)

https://ezto.mheducation.com/hm.tpx 76/98
12/16/21, 1:39 PM Assignment Print View

 
 148.    

(Appendix 13A) What is the present value of the net operating cash flows (sales minus cash
operating expenses) arising from the rebuilding and sale of the alternators, rounded to the nearest
dollar? (Do not round your intermediate calculations.)

 $577,864

 $582,735

 $591,884

 $476,454

Calculated using the NPV formula of Microsoft Excel.

 
 149.    

(Appendix 13A) What is the net present value of ALL cash flows associated with this investment,
rounded to the nearest dollar? (Do not round your intermediate calculations.)

 $362,950

 $377,864

 $279,819

 $391,884

Calculated using the NPV formula of Microsoft Excel.

https://ezto.mheducation.com/hm.tpx 77/98
12/16/21, 1:39 PM Assignment Print View

Westland College has a telephone system that is in poor condition. The system either can be
overhauled or replaced with a new system. The following data have been gathered concerning
these two alternatives:
 
  Present System Proposed New System
Purchase Cost New $250,000 $300,000
Accumulated Depreciation $240,000  
Overhaul costs needed now $230,000  
Annual Cash operating costs $180,000 $170,000
Salvage value now $160,000 $165,000
Salvage value at the end of $152,000 $200,000
eight years
Working capital required    

Westland College uses a 10% discount rate and the total - cost approach to capital budgeting
analysis. Both alternatives are expected to have a useful life of eight years. (Ignore income taxes in
this problem.)

 
 150.    

(Appendix 13A) What is the net present value of the alternative of overhauling the present system?
(Do not round your intermediate calculations and round your final answer to the nearest whole
number.)

 ($1,035,406)

 ($1,190,287)

 ($1,119,378)

 $717,225

Calculated using the PV formula of Microsoft Excel.

https://ezto.mheducation.com/hm.tpx 78/98
12/16/21, 1:39 PM Assignment Print View

 
 151.    

(Appendix 13A) What is the net present value of the alternative of purchasing the new system? (Do
not round your intermediate calculations and round your final answer to the nearest whole number.)

 ($1,236,495)

 ($1,169,963)

 ($1,076,662)

 ($969,895)

Calculated using the PV formula of Microsoft Excel.

Lambert Manufacturing has $60,000 to invest in either Project A or Project B. The following data are
available on these projects:
 
  Project A Project B
Cost of equipment needed now $120,000 $70,000
Working capital investment   $50,000
needed now
Annual net operating cash $50,000 $45,000
inflows
Salvage value of equipment in $15,000  
six years

Both projects have a useful life of six years. At the end of six years, the working capital investment
will be released for use elsewhere. Lambert's discount rate is 14%. (Ignore income taxes in this
problem.)

https://ezto.mheducation.com/hm.tpx 79/98
12/16/21, 1:39 PM Assignment Print View

 
 152.    

(Appendix 13A) The net present value of Project A is closest to which of the following? (Do not
round your intermediate calculations.)

 $67,610

 $74,450

 $81,267

 $141,267

Calculated using the NPV formula of Microsoft Excel.

 
 153.    

(Appendix 13A) The net present value of Project B is closest to which of the following? (Do not
round your intermediate calculations.)

 $54,990

 $77,769

 $57,268

 $127,805

Calculated using the NPV formula of Microsoft Excel.

https://ezto.mheducation.com/hm.tpx 80/98
12/16/21, 1:39 PM Assignment Print View

 
 154.    

Which of the following statements is(are) correct?

I. Project A is acceptable according to the net present value method.


II. Project A has an internal rate of return greater than 14%.

 I only.

 II only.

 Both I and II.

 Neither I nor II.

NPV > 0 so IRR > 14%.

Fast Food, Inc. has purchased a new donut maker. It cost $25,000 and has an estimated life of nine
years. The following annual donut sales and expenses are projected:
 
Sales   $30,000
Expenses:    
Flour, etc. (required in making $18,000  
donuts)
Salaries $7,000  
Depreciation $1,800 $26,800
Operating Income   $3,200

(Ignore income taxes in this problem.)

 
 155.    

The payback period on the new machine is closest to which of the following?

 1.4 years.

 2.7 years.

 3.6 years.

 5.0 years.

$25,000/ ($30,000 - $18,000 - $7,000) = 5 years.

https://ezto.mheducation.com/hm.tpx 81/98
12/16/21, 1:39 PM Assignment Print View

 
 156.    

The simple rate of return for the new machine is closest to which of the following?

 11.20%

 12.80%

 37.50%.

 80.00%.

$3,200 / $25,000 = 12.80%.

Purvell Company has just acquired a new machine. Data on the machine follow:
 
Purchase Cost $80,000
Annual cost savings $20,000
Life of the Machine 10 years

The company uses straight-line depreciation and a $5,000 salvage value. (The company considers
salvage value in making depreciation deductions.) Assume cash flows occur uniformly throughout a
year. (Ignore income taxes in this problem.)

 
 157.    

The payback period would be closest to which of the following?

 2.90 years.

 3.00 years.

 3.33 years.

 4.00 years.

$80,000/$20,000 = 4 years.

https://ezto.mheducation.com/hm.tpx 82/98
12/16/21, 1:39 PM Assignment Print View

 
 158.    

The simple rate of return would be closest to which of the following?

 12.50%

 17.50%

 15.63%

 30.00%

($20,000 - ($80,000 - $5,000)/ 10) / $80,000 = 15.63%.

Legend Company has purchased a machine for $190,000 that will be depreciated on the straight-
line basis over a ten-year period with no salvage value. The related cash flow from operations is
expected to be $38,000 a year. These cash flows from operations occur uniformly throughout the
year. (Ignore income taxes in this problem.)

 
 159.    

What is the payback period for this investment? (Round your answer to the one decimal place.)

 2.1 years.

 5.0 years.

 2.8 years.

 4.2 years.

$190,000 / $38,000 = 5 years.

https://ezto.mheducation.com/hm.tpx 83/98
12/16/21, 1:39 PM Assignment Print View

 
 160.    

What is the simple rate of return on the initial investment?

 10%

 24%

 28%

 36%

($38,000 - $190,000/10) / 190,000 = 10%.

Gates Company had taxable cash sales of $500,000 in Year 1. Tax-deductible cash expenses in Year
1 were $200,000, and capital cost allowance (CCA) deductions were $80,000. The income tax rate
was 25%.

 
 161.    

(Appendix 13B) What were the tax savings from the CCA tax shield for Year 1?

 $20,000

 $32,000

 $36,000

 $48,000

$80,000 * 0.25 = $20,000.

https://ezto.mheducation.com/hm.tpx 84/98
12/16/21, 1:39 PM Assignment Print View

 
 162.    

(Appendix 13B) What was the after-tax net cash inflow from all sources for Year 1?

 $245,000

 $156,000

 $180,000

 $216,000

($500,000 - $200,000) * (1-0.25) + $20,000 = $245,000.

River Company bought $70,000 worth of office equipment at the beginning of Year 1. This
equipment has a useful life of ten years and a salvage value at the end of its useful life of $10,000.
This equipment is in Class 7 with capital cost allowance (CCA) rate of 15%. The income tax rate is
40%.

 
 163.    

(Appendix 13B) What is the maximum amount of CCA that the company can deduct for tax purposes
for Year 1?

 $5,250

 $3,000

 $5,400

 $6,000

$70,000 * 0.5 * 0.15 = $5,250.

https://ezto.mheducation.com/hm.tpx 85/98
12/16/21, 1:39 PM Assignment Print View

 
 164.  
 

(Appendix 13B) What is the maximum amount of CCA that the company will be able to deduct for
tax purposes for Year 2? (Round your final answer to nearest whole dollar).

 $4,995

 $5,100

 $9,713

 $6,000

($70,000 - $5,250) * 0.15 = $9,713.

Alpine Company is analyzing two investment projects: P and Q. The following data are available:
 
  Project P Project Q
Investment in Computer $140,000 $0
Equipment now
Investment in Working Capital $0 $90,000
Now
Net Annual Operating Cash $20,000 $16,000
inflows
Life of Project 8 years 8 years

The computer equipment for Project P will have a total salvage value of $8,000 at end of eight
years. It will belong to Class 10 with a 30% maximum CCA rate. At the end of eight years, the working
capital for Project Q will be released for use elsewhere. The income tax rate is 30% and Alpine's
after-tax cost of capital is 10%.

https://ezto.mheducation.com/hm.tpx 86/98
12/16/21, 1:39 PM Assignment Print View

 
 165.    

(Appendix 13B) What is the approximate present value of the tax savings (for all years) due to the
CCA tax shield for Project P? (Do not round your intermediate calculations and round your final
answer to the nearest whole number.)

 $29,228

 $30,068

 $30,908

 $31,500

[(140,000 * .30 * .30)/(.30 + .10)] * [(1 + .5 * .10)/1.10] - [(8,000 * .30 * .30)/(.30 + .10)] * 1/(1.10)8 = $29,228.

 
 166.    

(Appendix 13A and 13B) What is the approximate present value of the after-tax net annual operating
cash inflows for Project P? (Do not round your intermediate calculations.)

 $32,010

 $53,074

 $74,689

 $106,699

20,000 * (1 -.30) = $14,000, then Calculated using the PV formula of Microsoft Excel.

Manti Company purchased a new machine on January 2, Year 1. Cost and other data relating to the
machine follow:
 
Cost of Machine $140,000
Salvage Value $20,000
Useful Life 12 years

The machine is in Class 7 with a maximum 15% CCA rate. Manti uses an after-tax discount rate of 12%
for capital budgeting decisions. The income tax rate is 40%.

https://ezto.mheducation.com/hm.tpx 87/98
12/16/21, 1:39 PM Assignment Print View

 
 167.    

(Appendix 13A and 13B) If Manti deducts the maximum CCA for tax purposes, what will be the
approximate present value (as of January 2, Year 1) of the CCA tax shield for Year 1?(Do not round
your intermediate calculations.)

 $3,750

 $4,200

 $6,429

 $7,500

140,000 * .5 * .15 * .40 = $4,200, then Calculated using the PV formula of Microsoft Excel.

 
 168.  
 

(Appendix 13A and 13B) If Manti deducts the maximum CCA for tax purposes, what will be the
approximate present value (as of January 2, Year 1) of the CCA tax shield for Year 2?(Do not round
your intermediate calculations.)

 $6,194

 $7,700

 $9,291

 $19,425

(140,000 - 140,000 * .5 * .15) * .15 * .40 = $7,770, then Calculated using the PV formula of Microsoft
Excel.

https://ezto.mheducation.com/hm.tpx 88/98
12/16/21, 1:39 PM Assignment Print View

Jackson Company is replacing an old delivery van with a new van. The following data relate to this
investment decision:
 
Cost of the new van now $17,000
Annual cash operating costs of the new Van $6,600
Useful life of the new van 6 years
Salvage value of the van at the End of six years $1,000
Original cost of the old van two years ago $18,000
Book value of the old van now $5,000
Salvage value of the old van now $1,000
Salvage value of the old van six year from now $500
Annual Cash operating Costs of the old van $8,000

The old van is in Class 10 with a maximum CCA rate of 30% and will last for six more years. The new
van is also in Class 10 with a maximum CCA rate of 30%. The income tax rate is 20%, and the
company's after-tax cost of capital is 10%.

 
 169.    

What is the incremental cash outlay now for the purchase of the new van?

 $17,000

 $16,000

 $20,000

 $21,000

$17,000 - $1,000 = $16,000.

https://ezto.mheducation.com/hm.tpx 89/98
12/16/21, 1:39 PM Assignment Print View

 
 170.  
 

(Appendix 13B) What is the approximate annual after-tax savings in cash operating costs?

 $400

 $1,120

 $1,200

 $2,000

($8,000 - $6,600) * (1-0.2) = $1,120.

 
 171.  
 

(Appendix 13A and 13B) What is the approximate present value of the after-tax net savings in cash
operating costs for all the years? (Do not round your intermediate calculations.)

 $3,484

 $4,878

 $8,711

 $12,000

Calculated using present value table for an annuity;


4.3553 * $1,120 = $4,878.

https://ezto.mheducation.com/hm.tpx 90/98
12/16/21, 1:39 PM Assignment Print View

 
 172.    

(Appendix 13B) What is the value of the incremental UCC (the "C" in the PV CCA formula) used in
the calculation for the present value of CCA tax savings?

 $20,000

 $16,000

 $18,000

 $26,000

$17,000 - $1,000 = $16,000.

A piece of equipment, acquired in Year 1, belongs to Class 7 with a maximum CCA rate of 15%. The
income tax rate is 45%. The tax savings (before discounting) from the CCA tax shield were $3,000
for Year two. The after-tax cost of capital is 8%.

 
 173.    

(Appendix 13B) What is the approximate undepreciated capital cost (UCC) balance for the
equipment at the beginning of Year 3? (Round your final answer to the nearest whole dollar).

 $7,076

 $44,444

 $31,450

 $47,175

$3,000/ (0.15 * 0.45) = $44,444.

https://ezto.mheducation.com/hm.tpx 91/98
12/16/21, 1:39 PM Assignment Print View

 
 174.    

(Appendix 13A and 13B) What was the present value of the tax savings from the CCA tax shield in
year 2 to the nearest dollar? (Round your final answer to the nearest whole dollar).

 $1,132

 $1,698

 $2,572

 $2,831

Calculated using the PV table;


0.8573 * $3,000 = $2,572.

Eureka Company is considering replacing an old computer with a new computer. The following data
relate to this investment decision:
 
Cost of the new computer $40,000
Annual cash operating costs of the new $19,000
computer
Working capital needed now for the new $1,000
computer
Useful life of the new computer 6 Years
Salvage value of the new computer at the end of $3,000
six years
Original cost of the old van two years ago $29,000
Salvage value of the old computer now $4,000
Salvage value of the old computer six years from $0
now

The new computer will belong to Class 10 with a maximum CCA rate of 30%. The income tax rate is
30% and the company's after-tax cost of capital is 12%.

https://ezto.mheducation.com/hm.tpx 92/98
12/16/21, 1:39 PM Assignment Print View

 
 175.  
 

What is the present value of the before-tax proceeds that will be received on the sale of the old
computer?

 $0

 $1,200

 $2,800

 $4,000

$4,000 * 100% = $4,000.

 
 176.    

(Appendix 13B) What is the approximate present value of the tax savings for all years because of
the CCA tax shield? (Do not round your intermediate calculations and round your final answer to the
nearest whole dollar.)

 $7,786

 $6,975

 $8,245

 $8,438

[(($40,000 - $4,000) *0.30 * 0.30)/(0.30 + 0.12)] * [(1 + 0.5 * 0.12)/1.12] - [($3,000 * 0.30 *0.30)/(0.30 +
0.12)] * 1.12-6 = $6,975.32 = $6,975 (rounded).

https://ezto.mheducation.com/hm.tpx 93/98
12/16/21, 1:39 PM Assignment Print View

 
 177.    

(Appendix 13A and 13B) What is the approximate present value of the after-tax net annual cash
operating inflows for all years? (Round your final answers to the nearest whole dollar).

 $12,334

 $23,024

 $54,682

 $41,114

$19,000 * (1 -0.30) = $13,300 then Calculated using the PV table to calculate an annuity.

 
 178.  
 

(Appendix 13A and 13B) What is the approximate present value of the after-tax non-operating cash
inflows that will occur in Year 6? (Do not round your intermediate calculations and round your final
answer to the nearest whole dollar.)

 $2,027

 $1,773

 $2,077

 $2,533

($1,000 + $3,000) = $4,000 then Calculated using the PV table.

https://ezto.mheducation.com/hm.tpx 94/98
12/16/21, 1:39 PM Assignment Print View

 
 179.    

(Appendix 13A) What is the approximate effective cost now of the working capital component of the
investment decision? (Do not round your intermediate calculations and round your final answer to
the nearest whole dollar.)

 $0

 $493

 $2,000

 $3,014

$1,000 - PV of $1,000 Calculated using the PV table.

The Morgan Company has been awarded a six-year contract to provide repair service to a
commercial bus line. Morgan Company has gathered the following data associated with the items
needed for this contract:
 
Cost of the Special equipment needed now $300,00
Working capital needed now $80,000
Net annual operating cash inflows $120,000
Equipment maintenance overhaul at the End of $20,000
Four Years
Salvage Value of the Equipment in six Years $20,000

The special equipment is in Class 7 with a maximum 15% CCA rate. The income tax rate is 40%, and
Morgan's after-tax cost of capital is 14%. At the end of six years, the working capital will be released
for use elsewhere.

 
 180.    

What is the total cash outlay needed to be made now?

 $300,000

 $360,000

 $380,000

 $400,000

300,000 + 80,000 = $380,000.


https://ezto.mheducation.com/hm.tpx 95/98
12/16/21, 1:39 PM Assignment Print View

 
 181.  
 

(Appendix 13A and 13B) The present value of the after-tax cash from the sale of the equipment at
the end of six years is closest to which of the following? (Do not round your intermediate
calculations.)

 $0

 $3,645

 $5,467

 $9,112

Calculated using the PV formula of Microsoft Excel.

 
 182.  
 

(Appendix 13A) The effective cost to Morgan of the need for working capital on the contract is
closest to which of the following? (Do not round your intermediate calculations and round your final
answer to the nearest whole number.)

 $36,480

 $43,553

 $58,131

 $80,000

80,000 - PV of 80,000 Calculated using the PV formula of Microsoft Excel.

https://ezto.mheducation.com/hm.tpx 96/98
12/16/21, 1:39 PM Assignment Print View

 
 183.  
 

(Appendix 13A and 13B) The present value of the total after-tax net cash inflows (outflows),
excluding any CCA tax shield, in Year 4 is closest to which of the following? (Do not round your
intermediate.)

 ($7,105)

 $23,684

 $35,525

 $38,269

(120,000 - 20,000) * (1 -.40) = $60,000, then Calculated using the PV formula of Microsoft Excel.

 
 184.    

(Appendix 13B) Assume the special equipment will have a zero salvage value (instead of $20,000)
at the end of six years. The present value of the total tax savings for all years because of the CCA
tax shield is closest to which of the following? (Do not round your intermediate calculations.)

 $56,373

 $58,258

 $60,143

 $62,069

[(300,000 * .15 *.40)/(.15 + .14)] * [(1 + .5 * .14)/(1 + .14)] = $58,258.

https://ezto.mheducation.com/hm.tpx 97/98
12/16/21, 1:39 PM Assignment Print View

Apex Corp., is planning to buy a production machine costing $200,000. This machine's expected
useful life is five years, with no residual value. Apex uses a discount rate of 8% and has calculated
the following data pertaining to the purchase and operation of this machine:
 
Year Estimated Annual Net Cash Inflow
1 $100,000
2 $80,000
3 $10,000
4 $10,000
5 $10,000

(Ignore income taxes in this problem.)

 
 185.  
 

What is the payback period for this investment?

 3.50 years.

 2.75 years.

 3.00 years.

 4.00 years.

$200,000 - $100,000 - $80,000 - $10000 = $10,000;


Payback = 3 + $10,000 / $10,000 = 4 years

 
 186.  
 

(Appendix 13A) The net present value of this investment is closest to which of the following? (Do not
round your intermediate calculations.)

 ($16,726)

 $81,025

 $50,000

 $80,000

Calculated using the NPV formula of Microsoft Excel.


https://ezto.mheducation.com/hm.tpx 98/98

You might also like